LSAT and Law School Admissions Forum

Get expert LSAT preparation and law school admissions advice from PowerScore Test Preparation.

User avatar
 Dave Killoran
PowerScore Staff
  • PowerScore Staff
  • Posts: 5852
  • Joined: Mar 25, 2011
|
#41308
Complete Question Explanation
(The complete setup for this game can be found here: lsat/viewtopic.php?t=15710)

The correct answer choice is (D)

This question tests the limited scenarios that result when Z is added first. When Z is added first, M cannot be second, third, or last. Hence, we can create two hypotheticals based on the position of M:
J03_Game_#1_#5_diagram 1.png
Because the position of M is restricted, we should first apply the last rule (and the third rule because it affects K and T) :
J03_Game_#1_#5_diagram 2.png
Finally, by applying the second rule, we can fill in each hypothetical:
J03_Game_#1_#5_diagram 3.png
When applying the second rule, in Hypothetical #1 we can infer that O must be added fifth by taking the following steps:
  • 1. ..... There are only three open spaces.

    2. ..... Since K/T :longline: M, we know that K/T must be added second or third.

    3. ..... Since L :longline: O, L must then be second or third.

    4. ..... The only remaining space for O is the fifth.

Both hypotheticals must be considered when answering the question. Answer choice (D) is correct because in Hypothetical #1 we know M is fourth, and in Hypothetical #2 we know L cannot be fourth because L :longline: O. Thus, L cannot be added fourth when Z is added first.

Question #5 is probably the most difficult question, and the only effective way to attack the question is to use hypotheticals. Hopefully, by quickly answering questions #1 through #4 you will build enough time to comfortably work through question #5.
You do not have the required permissions to view the files attached to this post.
 macbrae
  • Posts: 1
  • Joined: Jan 26, 2018
|
#43242
Where is the restriction on M coming from? Why can M not be 3rd?
I came up with the following order: Z K M T O L for this question.
 Emily Haney-Caron
PowerScore Staff
  • PowerScore Staff
  • Posts: 577
  • Joined: Jan 12, 2012
|
#43297
Hi macbrae,

The M restriction here is a tricky inference to spot, so great question! It's a combination of rules 1 and 2. If L is before O (rule 2), L can't be last (rule 1), so if Z is first, then M can't be third.

See if you can take it from there, and let us know if you have any other questions! :)
 jessamynlockard
  • Posts: 42
  • Joined: Jan 15, 2018
|
#44533
In Lesson 3HW, pg 3-57, Game #6, June 2003:

If rule 3 says that neither the tomatoes nor the kale is added fifth, why can't we infer a not law saying M cannot be 4?

This comes up as I go to answer Question 5.
 Adam Tyson
PowerScore Staff
  • PowerScore Staff
  • Posts: 5153
  • Joined: Apr 14, 2011
|
#44550
Thanks for the question, Jessamyn! I have moved your post to the discussion of Question 5 in that first game from June 2003

To expand on the discussion here, and on the general rules of the game, let me turn the question around on you: Why WOULD we infer that M cannot be 4th? What problem would that create? We know that M is between K and T somewhere, per the last rule. If M was 4th, and neither K nor T was 5th, how would we satisfy that last rule? We would have to put one of K or T 6th, and the other one somewhere before M. For question 5, that means putting one of K or T either 2nd or 3rd. Play with that idea and see what else comes up! If the discussion threads for that game don't answer your question to your satisfaction, reply in that thread requesting further assistance and we will jump all over it!
 Katya W
  • Posts: 42
  • Joined: Dec 03, 2019
|
#74396
If I didn’t infer the restrictions of M’s placement (except that it obviously cannot be 2nd), and I didn’t know to link rules 1 and 2 from the beginning, how would I attack the problem? Thank you
 Jeremy Press
PowerScore Staff
  • PowerScore Staff
  • Posts: 1000
  • Joined: Jun 12, 2017
|
#74460
Hi Katya,

Without first having seen that the first and second rules are connected through the contrapositive of the first rule, the only possible way to attack the question is to test every answer choice individually and see whether it could be true (what some people call "plug and chug," i.e. plug the answer choice into a mini-diagram and chug through a solution). That's an inevitably slow process. It's probably not worth it to do that for question 5 of the section, since on this section you had 18 more questions remaining, many of which are much easier than this question. However, it is a reliable method that will eventually get you to the right answer.

This question is a great illustration of how important it is to find inferences at the start of the game, since they will save you time on difficult questions like this one. Take your time to think about how you could've easily spotted that inference (first by noticing the common variable in the first two rules, then by diagramming the contrapositives of those rules, and looking for connections that arise). Try to remember for any future games that have two different conditional rules that share a variable, you might be able to create a connection through a contrapositive. That'll put you on the best path to both speed and accuracy!

I hope this helps!

Jeremy
 Katya W
  • Posts: 42
  • Joined: Dec 03, 2019
|
#74637
Thank you Jeremy! Great explanation!
User avatar
 Olive123
  • Posts: 1
  • Joined: Jan 10, 2021
|
#83128
Why are K/T placed in the second block for both hypotheticals? Why isn’t L placed there?
User avatar
 KelseyWoods
PowerScore Staff
  • PowerScore Staff
  • Posts: 1079
  • Joined: Jun 26, 2013
|
#83225
Hi Olive!

When we use parentheses on our diagrams, it means that we know which variables go in those slots, but we don't know the exact order. This helps us diagram more efficiently so that we can see the possibilities without diagramming out every single exact order.

So in Hypothetical #1, we have the parentheses around (K/T, L) over slots 2 and 3. That means in slots 2 and 3 we could have K L, T L, L K, or L T. In Hypothetical #2, we have parentheses around (K/T, L :longline: O) over slots 2, 3, and 4. That means that slots 2, 3, and 4 could be: K L O, T L O, L K O, L T O, L O K, or L O T.

Hope this helps!

Best,
Kelsey

Get the most out of your LSAT Prep Plus subscription.

Analyze and track your performance with our Testing and Analytics Package.